confusing cr

This topic has expert replies
Master | Next Rank: 500 Posts
Posts: 183
Joined: Wed Feb 09, 2011 3:08 am
Location: Delhi
Thanked: 1 times
Followed by:5 members

confusing cr

by alltimeacheiver » Thu Feb 10, 2011 2:00 am
Lightbox, Inc., owns almost all of the movie theaters in Washington County and has
announced plans to double the number of movie screens it has in the county within five
years. Yet attendance at Lightbox's theaters is only just large enough for profitability
now and the county's population is not expected to increase over the next ten years.
Clearly, therefore, if there is indeed no increase in population, Lightbox's new screens
are unlikely to prove profitable.
Which of the following, if true about Washington County, most seriously weakens the
argument?
A. Though little change in the size of the population is expected, a pronounced shift
toward a younger, more affluent, and more entertainment-oriented population is
expected to occur.
23
B. The sales of snacks and drinks in its movie theaters account for more of
Lightbox's profits than ticket sales do.
C. In selecting the mix of movies shown at its theaters, Lightbox's policy is to avoid
those that appeal to only a small segment of the moviegoing population.
D. Spending on video purchases, as well as spending on video rentals, is currently no
longer increasing.
E. There are no population centers in the county that are not already served by at
least one of the movie theaters that Lightbox owns and operates.

User avatar
Master | Next Rank: 500 Posts
Posts: 436
Joined: Tue Feb 08, 2011 3:07 am
Thanked: 72 times
Followed by:6 members

by manpsingh87 » Thu Feb 10, 2011 2:25 am
alltimeacheiver wrote:Lightbox, Inc., owns almost all of the movie theaters in Washington County and has
announced plans to double the number of movie screens it has in the county within five
years. Yet attendance at Lightbox's theaters is only just large enough for profitability
now and the county's population is not expected to increase over the next ten years.
Clearly, therefore, if there is indeed no increase in population, Lightbox's new screens
are unlikely to prove profitable.
Which of the following, if true about Washington County, most seriously weakens the
argument?
A. Though little change in the size of the population is expected, a pronounced shift
toward a younger, more affluent, and more entertainment-oriented population is
expected to occur.
23
B. The sales of snacks and drinks in its movie theaters account for more of
Lightbox's profits than ticket sales do.
C. In selecting the mix of movies shown at its theaters, Lightbox's policy is to avoid
those that appeal to only a small segment of the moviegoing population.
D. Spending on video purchases, as well as spending on video rentals, is currently no
longer increasing.
E. There are no population centers in the county that are not already served by at
least one of the movie theaters that Lightbox owns and operates.
IMO A..!!!!

Master | Next Rank: 500 Posts
Posts: 183
Joined: Wed Feb 09, 2011 3:08 am
Location: Delhi
Thanked: 1 times
Followed by:5 members

by alltimeacheiver » Thu Feb 10, 2011 2:46 am
why not b becoz it is taking about the profits in conclusion. so we target conclusion then we can target aletrnative way of profit making... pls reply

User avatar
Senior | Next Rank: 100 Posts
Posts: 71
Joined: Wed Oct 14, 2009 12:00 pm
Location: New Jersey, US
Thanked: 2 times

by saurabh_maths » Thu Feb 10, 2011 3:32 am
IMO it shd be A.

A- it tells that there is other reason for increase in sale of tickets even though the population is not going to increase.That said if a greater segment for e.g kids are becoming adolescents and are prone to entertainment via movies, then this could lead to increase in profits. Hence this is correct ans.

B- It talks abt sale of snacks inside movie theaters. Q is abt the addition of movie screens. Nothing has been said about people buying more snacks. so incorrect.

C - this was a contender but still it does not imply anything about the increase in population coming to movie theaters.So even it may be true , it might ot increase the profitabilty if number of ppl don increase.So incorrect.

D out of scope

E- infact it strengthen little bit. so incorrect.

What the OA ??

User avatar
Master | Next Rank: 500 Posts
Posts: 436
Joined: Tue Feb 08, 2011 3:07 am
Thanked: 72 times
Followed by:6 members

by manpsingh87 » Thu Feb 10, 2011 3:35 am
alltimeacheiver wrote:why not b becoz it is taking about the profits in conclusion. so we target conclusion then we can target aletrnative way of profit making... pls reply
To understand the given premise "Yet attendance at Lightbox's theaters is only just large enough for profitability
now and the county's population is not expected to increase over the next ten years."

lets assume that county's population is 5 and no. of theaters are also 5 of which lightbox owns 4, so if in the future lightbox increase its movie theater to 8 then it will definitely make loss irrespective of the profit that it is making by selling snacks in its theater..because out of 8 theaters 4 will be making profit whereas rest 4 will be making loss..!!! i hope it helps..!!!